Difference between revisions of "2013 AMC 12A Problems/Problem 23"
(→Video Solution by Richard Rusczyk) |
|||
(9 intermediate revisions by 6 users not shown) | |||
Line 7: | Line 7: | ||
==Solution== | ==Solution== | ||
− | We first note that diagonal <math> \overline{AC} </math> is of length <math> \sqrt{6} + \sqrt{2} </math>. It must be that <math> \overline{AP} </math> divides the diagonal into two segments in the ratio <math>\sqrt{3}</math> to <math>1</math>. It is not difficult to visualize that when the square is rotated, the initial and final squares overlap in a rectangular region of dimensions <math>2</math> by <math>\sqrt{3} + 1</math>. The area of the overall region (of the initial and final squares) is therefore twice the area of the original square minus the overlap, or <math> 2 (\sqrt{3} + 1)^2 - 2 (\sqrt{3} + 1) = 2 (4 + 2 \sqrt{3}) - 2 \sqrt{3} - 2 = 6 + 2 \sqrt{3} </math>. | + | We first note that diagonal <math> \overline{AC} </math> is of length <math> \sqrt{6} + \sqrt{2} </math>. It must be that <math> \overline{AP} </math> divides the diagonal into two segments in the ratio <math>\sqrt{3}</math> to <math>1</math>. It is not difficult to visualize that when the square is rotated, the initial and final squares overlap in a rectangular region of dimensions <math>2\sqrt{3}</math> by <math>\sqrt{3} + 1</math>. The area of the overall region (of the initial and final squares) is therefore twice the area of the original square minus the overlap, or <math> 2 (\sqrt{3} + 1)^2 - 2 (\sqrt{3} + 1) = 2 (4 + 2 \sqrt{3}) - 2 \sqrt{3} - 2 = 6 + 2 \sqrt{3} </math>. |
− | + | The area also includes <math>4</math> circular segments. Two are quarter-circles centered at <math>P</math> of radii <math>\sqrt{2}</math> (the segment bounded by <math> \overline{PA} </math> and <math> \overline{PA'}</math>) and <math>\sqrt{6}</math> (that bounded by <math>\overline{PC}</math> and <math>\overline{PC'}</math>). Assuming <math>A</math> is the bottom-left vertex and <math>B</math> is the bottom-right one, it is clear that the third segment is formed as <math>B</math> swings out to the right of the original square [recall that the square is rotated counterclockwise], while the fourth is formed when <math>D</math> overshoots the final square's left edge. To find these areas, consider the perpendicular from <math>P</math> to <math>\overline{BC}</math>. Call the point of intersection <math>E</math>. From the previous paragraph, it is clear that <math>PE = \sqrt{3}</math> and <math>BE = 1</math>. This means <math>PB = 2</math>, and <math>B</math> swings back inside edge <math>\overline{BC}</math> at a point <math>1</math> unit above <math>E</math> (since it left the edge <math>1</math> unit below). The triangle of the circular sector is therefore an equilateral triangle of side length <math>2</math>, and so the angle of the segment is <math>60^{\circ}</math>. Imagining the process in reverse, it is clear that the situation is the same with point <math>D</math>. | |
− | + | The area of the segments can be found by subtracting the area of the triangle from that of the sector; it follows that the two quarter-segments have areas <math> \frac{1}{4} \pi (\sqrt{2})^2 - \frac{1}{2} \sqrt{2} \sqrt{2} = \frac{\pi}{2} - 1 </math> and <math> \frac{1}{4} \pi (\sqrt{6})^2 - \frac{1}{2} \sqrt{6} \sqrt{6} = \frac{3 \pi}{2} - 3 </math>. The other two segments both have area <math>\frac{1}{6} \pi (2)^2 - \frac{(2)^2 \sqrt{3}}{4} = \frac{2 \pi}{3} - \sqrt{3} </math>. | |
− | The area | + | The total area is therefore <cmath>(6 + 2 \sqrt{3}) + (\frac{\pi}{2} - 1) + (\frac{3 \pi}{2} - 3) + 2 (\frac{2 \pi}{3} - \sqrt{3})</cmath> <cmath>= 2 + 2 \sqrt{3} + 2 \pi + \frac{4 \pi}{3} - 2 \sqrt{3}</cmath> <cmath>= \frac{10 \pi}{3} + 2</cmath> <cmath>= \frac{1}{3} (10 \pi + 6) </cmath> |
− | + | Since <math>a = 10</math>, <math>b = 6</math>, and <math>c = 3</math>, the answer is <math>a + b + c = 10 + 6 + 3 = \boxed{\textbf{(C)} \ 19}</math>. | |
+ | ==Video Solution by Richard Rusczyk== | ||
+ | https://artofproblemsolving.com/videos/amc/2013amc12a/362 | ||
− | + | ~dolphin7 | |
== See also == | == See also == | ||
{{AMC12 box|year=2013|ab=A|num-b=22|num-a=24}} | {{AMC12 box|year=2013|ab=A|num-b=22|num-a=24}} | ||
+ | |||
+ | [[Category:Introductory Geometry Problems]] | ||
+ | [[Category:Area Problems]] | ||
+ | {{MAA Notice}} |
Latest revision as of 15:51, 3 April 2020
Problem
is a square of side length . Point is on such that . The square region bounded by is rotated counterclockwise with center , sweeping out a region whose area is , where , , and are positive integers and . What is ?
Solution
We first note that diagonal is of length . It must be that divides the diagonal into two segments in the ratio to . It is not difficult to visualize that when the square is rotated, the initial and final squares overlap in a rectangular region of dimensions by . The area of the overall region (of the initial and final squares) is therefore twice the area of the original square minus the overlap, or .
The area also includes circular segments. Two are quarter-circles centered at of radii (the segment bounded by and ) and (that bounded by and ). Assuming is the bottom-left vertex and is the bottom-right one, it is clear that the third segment is formed as swings out to the right of the original square [recall that the square is rotated counterclockwise], while the fourth is formed when overshoots the final square's left edge. To find these areas, consider the perpendicular from to . Call the point of intersection . From the previous paragraph, it is clear that and . This means , and swings back inside edge at a point unit above (since it left the edge unit below). The triangle of the circular sector is therefore an equilateral triangle of side length , and so the angle of the segment is . Imagining the process in reverse, it is clear that the situation is the same with point .
The area of the segments can be found by subtracting the area of the triangle from that of the sector; it follows that the two quarter-segments have areas and . The other two segments both have area .
The total area is therefore
Since , , and , the answer is .
Video Solution by Richard Rusczyk
https://artofproblemsolving.com/videos/amc/2013amc12a/362
~dolphin7
See also
2013 AMC 12A (Problems • Answer Key • Resources) | |
Preceded by Problem 22 |
Followed by Problem 24 |
1 • 2 • 3 • 4 • 5 • 6 • 7 • 8 • 9 • 10 • 11 • 12 • 13 • 14 • 15 • 16 • 17 • 18 • 19 • 20 • 21 • 22 • 23 • 24 • 25 | |
All AMC 12 Problems and Solutions |
The problems on this page are copyrighted by the Mathematical Association of America's American Mathematics Competitions.